LSAT and Law School Admissions Forum

Get expert LSAT preparation and law school admissions advice from PowerScore Test Preparation.

User avatar
 Dave Killoran
PowerScore Staff
  • PowerScore Staff
  • Posts: 5972
  • Joined: Mar 25, 2011
|
#26444
Complete Question Explanation
(The complete setup for this game can be found here: lsat/viewtopic.php?t=8639)

The correct answer choice is (C)


Apply the rules and inferences in this order: inference that H must be first, fifth rule, second rule, and finally, the first, third, and fourth rules can be applied in any order since they are roughly equivalent in form.

Answer choice (A) is incorrect because H must be more popular than J.

Answer choice (B) is incorrect because J must be more popular than Q.

Answer choice (C) is the correct answer choice.

Answer choice (D) is incorrect because L must be more popular than V.

Answer choice (E) is incorrect because S cannot be seventh.
 svazquez88
  • Posts: 1
  • Joined: Jan 14, 2024
|
#105441
So I got the correct answer of (C) by simply going through each answer and using process of elimination as each answer choice violated the set of rules. What I found a little confusing was when I read Dave’s explanation to the answer regarding inferences, namely the inference that H is must be first.

Can you please help me see that inference in this question.
User avatar
 EmilyOwens
PowerScore Staff
  • PowerScore Staff
  • Posts: 27
  • Joined: Feb 27, 2024
|
#105457
svazquez88 wrote: Sun Feb 25, 2024 7:05 pm So I got the correct answer of (C) by simply going through each answer and using process of elimination as each answer choice violated the set of rules. What I found a little confusing was when I read Dave’s explanation to the answer regarding inferences, namely the inference that H is must be first.

Can you please help me see that inference in this question.

Hi S,

Happy to help! Coming to the conclusion that H must be first involves creating a chain relationship between the rules.

Combining rules 1 and 2, we can deduce that H must come before J, L, and Q. This is because rule 1 states H must come before J and L, and rule 2 creates a chain relationship with the variable the two rules have in common: J. Combining the two rules looks like this:
IMG_5176.jpeg
Rule 3 also creates a chain relationship because rule 1 and rule 3 share a common variable: L. That, combined with the chain relationship we created from rules 1 and 2, comes together to look like this:
IMG_5175.jpeg
Rule 4 tells us P and S must come after Q, which already comes after both H and J. This can be combined with our current setup to create the following:

IMG_5177.jpeg
We see that, from combining these rules together, all programs must follow H in popularity, making H the only variable that can go in spot 1.

I hope this helps! :)
You do not have the required permissions to view the files attached to this post.

Get the most out of your LSAT Prep Plus subscription.

Analyze and track your performance with our Testing and Analytics Package.